The Student Room Group

Edexcel A2 C4 Mathematics June 2016 - Official Thread

Scroll to see replies

And since we have only 2 weeks left reckon doing most Solomon papers all gold papers and all papers from the last 2 years including Jan ial should be enough


Posted from TSR Mobile
Original post by Supermanxxxxxx
ImageUploadedByStudent Room1464640913.197247.jpg
Spent so long on a still couldn't do it


Posted from TSR Mobile


Did you divide through by (X-6)(X-3) then do partial fractions to put it as A/(x-6) + B/(X-3) ?
Original post by Middriver
Did you divide through by (X-6)(X-3) then do partial fractions to put it as A/(x-6) + B/(X-3) ?


Nope I didn't even think of that, thanks


Posted from TSR Mobile
Original post by Supermanxxxxxx
ImageUploadedByStudent Room1464640913.197247.jpg
Spent so long on a still couldn't do it


Posted from TSR Mobi

Edit:nevermind
Original post by Clovers
Hi,
Can someone please explain why, when solving a differential equation, you don't put constants of integration on both sides? Thanks in advance :smile:

Posted from TSR Mobile


hi look at this video to understand it http://www.examsolutions.net/maths-revision/core-maths/integration/differential-equations/seperable/tutorial-1a.php
Original post by Supermanxxxxxx
Nope I didn't even think of that, thanks


Posted from TSR Mobile


yh use partial fractions
Anybody know how I should go about learning vectors, I think I understand the topic but can't do any pp questions! :frown:
Original post by metrize
Can someone help on jan 13 c4 question 8?

https://googledrive.com/host/0B1ZiqBksUHNYQXE5T2xiNDBRd2s/January%202013%20QP%20-%20C4%20Edexcel.pdf

This is my method:
-ln(3-theta all over A)=t/125
So 3-theta over A=e^(-t/125)
So 3-theta = Ae^(-t/125)
So theta = 3-Ae^(-t/125)


But the answer is meant to be theta = 3+Ae^(-t/125)


Not sure why you start with -ln(3-theta all over A) I thought the A would be a constant on the dt side, so it would start -ln(3-theta)= t/125 +c ?
Original post by Middriver
Not sure why you start with -ln(3-theta all over A) I thought the A would be a constant on the dt side, so it would start -ln(3-theta)= t/125 +c ?


Yeah i called the constant ln A so then when i took it away and with logs rules i made it -ln( 3-theta all over A)
Original post by metrize
Yeah i called the constant ln A so then when i took it away and with logs rules i made it -ln( 3-theta all over A)


Oh right, did you manage to get it right?
guys for c3, when drawing modulus graphs and equating it to eachother how would you know if they intercept twice or not?

https://07a69ccf283966549a9350d1a66951a7bc96e2dc.googledrive.com/host/0B1ZiqBksUHNYZ0JQM1NRcmdHdXM/June%202014%20(R)%20QP%20-%20C3%20Edexcel.pdf

heres an example, question 7
Original post by Middriver
Oh right, did you manage to get it right?


I did the steps I had said at first but fot the signs wrong in the end so im not sure where i went wrong
Original post by metrize
I did the steps I had said at first but fot the signs wrong in the end so im not sure where i went wrong


Hmh shall I send my workings might be able to see where sign change went wrong? I didn't use lnA though so don't want to confuse you or anything.
Original post by Middriver
Hmh shall I send my workings might be able to see where sign change went wrong? I didn't use lnA though so don't want to confuse you or anything.


Yeah please do, thanks
Original post by metrize
Can someone help on jan 13 c4 question 8?

https://googledrive.com/host/0B1ZiqBksUHNYQXE5T2xiNDBRd2s/January%202013%20QP%20-%20C4%20Edexcel.pdf

This is my method:
-ln(3-theta all over A)=t/125
So 3-theta over A=e^(-t/125)
So 3-theta = Ae^(-t/125)
So theta = 3-Ae^(-t/125)


But the answer is meant to be theta = 3+Ae^(-t/125)


hello, your first step is wrong.. why are you dividing by A? thats not how its done.. the constant youre meant to be Adding is C and not A. Heres how i did it:

1) Dtheta/dt = (3-theta) / 125
2) 1/(3-theta) with respect to dtheta = 1/125 with respect to dt
3) -ln(3-theta) = t/125 + 'C'
4) ln(3-theta) = -t/125 - C
5) (3-theta) = e-0.008t-c
6) theta= 3 - e-0.008t-c
7) theta= 3 - (e-0.008t e-c )
8) theta = -e-c e-0.008t +3
9) theta =A e-0.008t +3
where A= -e-c it says A is a constant and -e-c is a constant
Original post by jovdawesome
hello, your first step is wrong.. why are you dividing by A? thats not how its done.. the constant youre meant to be Adding is C and not A. Heres how i did it:

1) Dtheta/dt = (3-theta) / 125
2) 1/(3-theta) with respect to dtheta = 1/125 with respect to dt
3) -ln(3-theta) = t/125 + 'C'
4) ln(3-theta) = -t/125 - C
5) (3-theta) = e-0.008t-c
6) theta= 3 - e-0.008t-c
7) theta= 3 - (e-0.008t e-c )
8) theta = -e-c e-0.008t +3
9) theta =A e-0.008t +3
where A= -e-c it says A is a constant and -e-c is a constant


I did that because at the beginning i said the constant is ln A and so i took away ln A and with log rules it divides by A inside the log, its the way that our teacher had taught it us, will learn how to do it with c, thanks for posting the working out
Original post by ryandaniels2015
Anybody know how I should go about learning vectors, I think I understand the topic but can't do any pp questions! :frown:

any help?
Nvm he's done it^
Original post by metrize
I did that because at the beginning i said the constant is ln A and so i took away ln A and with log rules it divides by A inside the log, its the way that our teacher had taught it us, will learn how to do it with c, thanks for posting the working out


When adding the constant, we must use 'C' in most cases. We must only add lnC if it makes it easier for us to solve it. For example if a question was ln(x-2)= ln5.. we can add lnC here because it will be easier for us to solve as we can cancel the ln on both sides. However in all questions ur simply meant to add C and NOT lnC..
I recommend you go over videos on examsolutions and learn this topic ASAP as we have just above 2 weeks left for the exam .. Good luck :smile:
Original post by jovdawesome
When adding the constant, we must use 'C' in most cases. We must only add lnC if it makes it easier for us to solve it. For example if a question was ln(x-2)= ln5.. we can add lnC here because it will be easier for us to solve as we can cancel the ln on both sides. However in all questions ur simply meant to add C and NOT lnC..
I recommend you go over videos on examsolutions and learn this topic ASAP as we have just above 2 weeks left for the exam .. Good luck :smile:


Oh wait i think my A is different to the A in the actuak exam question and that i might have gotten confused with the letters... so my -A is a constant which is just A as theyre both still constants and so can i change the sign of it at the end?

Quick Reply

Latest

Trending

Trending